PT73.4.19 and PT34.2.2 (please help)

Hi guys,

I am having trouble understanding JY's reasoning behind choosing correct answers in the above problems.
PT73.4.19 is a necessary assumption question and PT34.2.2 is a sufficient assumption question, and their premise - conclusion reasoning is essentially identical:

We should do A, so the author concludes that we should do B.

In PT34.2.2, the correct answer was (E), which says A -> B.
However in PT73.4.19, the correct answer was (B), which says, B helps A, which kind of sounds like the typical reversal answer choice. I understand how the correct answer choice was necessary for the argument to make sense, but if (B) said A helps B, would this be incorrect answer choice?

On a side note, how should I approach a conditional statement containing the word, "do"?
In PT34.2.2, JY draws a conditional diagram using "Do it," because the stimulus says "the city should always do what makes good economic sense," whereas in PT73.4.19 JY does not, even though the stimulus says "we must do what we can to prevent this loss of motivation." I understand either approach can lead to choosing the correct answer, but what should be the rule of thumb?

Any help would be appreciated.

Comments

  • LSATcantwinLSATcantwin Alum Member Sage
    edited November 2017 13286 karma

    This is a little bit difficult. There is a lot going on in what you are asking. Let's start with some basic stuff though.

    1.) What is a conditional statement at its core?

    • It consists of a sufficient and a necessary condition that are linked by a logical indicator.

    Some of the logical indicators are listed here: https://7sage.com/lesson/4-translation-groups-summary/

    Now let us address the word "DO". Do you think the word "Do" acts like a logical indicator? In other words, does the word "DO" signal the presence of a sufficient and necessary condition?

    • If I go to the store, then I buy milk. --- IF is our logical indicator here. It introduces the sufficient condition.

    Let's try that with the word "Do"

    Try and link the idea of "going to the store" with "buying milk" with the word do. Can you do that?

    Do is not one of our logical indicators and therefore it does not note something that can be translated into Lawgic.

    Now with the two questions you linked, we are tasked with finding a Sufficient Assumption in one and a Necessary assumption in the other. Do you recall the difference?

    • Sufficient Assumption - Makes the conclusion VALID. Recall J.Y.'s explanation of the different levels of an argument. Valid means it is an air tight argument. It means the conclusion now follows 100%.

    https://7sage.com/lesson/how-to-find-the-sufficient-assumption/

    • Necessary Assumption - Is an assumption that the argument has to make, or the conclusion goes to crap. If we take the necessary assumption away the entire conclusion is destroyed.

    https://7sage.com/lesson/how-to-find-necessary-assumptions/

    So we have to approach these questions differently.

    On PT73 our task is to find the Necessary assumption. So if we take this AC and negate it, the entire argument will fall apart. For these types of questions I very rarely use conditional lawgic. I try and read it looking for something that the argument takes for granted. (This is why J.Y. Didn't map this problem out. It's not based in conditional lawgic in any significant way)

    In this case AC B does just that.

    • Enabling people to believe that better futures are possible will help prevent the loss of motivation that results from pessimistic beliefs.

    Negated

    • Enabling people to believe that better futures are possible will NOT help prevent the loss of motivation that results from pessimistic beliefs.

    Do you see how the negation here destroyed his conclusion? He said we must enable our children to believe that better futures are possible in order to prevent loss of motivation. But if showing people that better futures are possible doesn't stop them from losing motivation, what good is his argument?

    Now PT34.2.2 is a sufficient assumption question. Many times these are filled to the brim with lawgic and conditional statements. (Hence why J.Y. mapped this out) However J.Y. mapped this because of the word "always" not the word "Do". Always is a group 2 indicator word.

    What our job here is, is to make the conclusion valid.

    The argument says.

    P1.) Having an efficent, attractive subway cars makes good economic sense.
    P2.) The city should ALWAYS do what makes good economic sense.

    C:) Therefore the city needs to purchase new subway cars.

    Okay we are missing something to make this valid. Subway cars are in the conclusion but no where else. We need to link them into the argument to make it so we need to buy them.

    Answer E does this.

    Answer Choice E.) New subway cars are required in order for the city to have a subway system that is efficient and attractive.
    P1.) Having an efficient, attractive subway cars makes good economic sense.
    P2.) The city should ALWAYS do what makes good economic sense.

    C:) Therefore the city needs to purchase new subway cars.

    Do you see how by adding E to the argument we can now say that new subway cars are required in order to have an efficient and attractive subway system. And since a subway system that is efficient and attractive makes good economic sense, the city must get one. So they need to buy subway cars in order to achieve this.

    Sorry this is long and convoluted. Feel free to PM me if you need more explanation. Your question was fairly loaded haha

  • BinghamtonDaveBinghamtonDave Alum Member 🍌🍌
    8689 karma

    Excellent work @LSATcantwin

  • jsfollowerjsfollower Member
    66 karma

    @LSATcantwin Hi, thanks for the response. Sorry, I meant like when the word "do" appears after the logical indicators like always and must, what should be the approach?

    In PT 34, the indicator was always, and in PT 73, it was must. I understand "do" alone is not a logical indicator.

    So, would it be correct to say these two questions had basically the same argument structures? and also can (B) in PT73 still be correct if the order was reversed as a necessary assumption answer choice? The reason I am asking this is because when I see a clear gap in necessary assumption questions, the correct answer is almost always the one that fills the gap. For example, the stimulus says A and concludes B, the correct answer is A -> B (which can also be a sufficient assumption at the same time). However, this answer choice was atypical as the order was reversed.

    Thanks again.

  • LSATcantwinLSATcantwin Alum Member Sage
    13286 karma

    @jsfollower said:
    @LSATcantwin Hi, thanks for the response. Sorry, I meant like when the word "do" appears after the logical indicators like always and must, what should be the approach?

    In PT 34, the indicator was always, and in PT 73, it was must. I understand "do" alone is not a logical indicator.

    So, would it be correct to say these two questions had basically the same argument structures? and also can (B) in PT73 still be correct if the order was reversed as a necessary assumption answer choice? The reason I am asking this is because when I see a clear gap in necessary assumption questions, the correct answer is almost always the one that fills the gap. For example, the stimulus says A and concludes B, the correct answer is A -> B (which can also be a sufficient assumption at the same time). However, this answer choice was atypical as the order was reversed.

    Thanks again.

    Ah I see what you are getting at. So a necessary assumption can be a sufficient assumption and vice versa. Both can be seen to fill the gap in an argument. The main distinction is that for a SA it must make our conclusion valid. NA doesn't have to do anything except strengthen the argument when it's added.

    I will respond in more depth when I get home. Sorry I can’t do it during my commute my reception goes away on the train haha

  • jsfollowerjsfollower Member
    66 karma

    @LSATcantwin Looking forward to your response!

  • LSATcantwinLSATcantwin Alum Member Sage
    13286 karma

    @jsfollower i have not forgotten! I was unexpectedly busy when I got home today. First thing tomorrow (7-8am) I will expand! Sorry!!!

  • LSATcantwinLSATcantwin Alum Member Sage
    edited November 2017 13286 karma

    Okay - I just watched J.Y's full explanation of PT73.4.19 and I'm not sure why it even went all that in depth haha. SO I am going to call in @BinghamtonDave again to help out with your question. I approached this question in a far different way than J.Y. did. I try to avoid "lawgic" every chance I get so I didn't map it like this at all.

    • I am not the best to answer this question! Sorry!!
  • jsfollowerjsfollower Member
    66 karma

    Anyone?

Sign In or Register to comment.